Rearrange The Terms First

Algebra Level 3

Find the minimum value of x 6 + y 6 54 x y x^6 + y^6 -54xy , where x x and y y are real numbers .


The answer is -108.

This section requires Javascript.
You are seeing this because something didn't load right. We suggest you, (a) try refreshing the page, (b) enabling javascript if it is disabled on your browser and, finally, (c) loading the non-javascript version of this page . We're sorry about the hassle.

4 solutions

Manuel Kahayon
Jun 10, 2016

Relevant wiki: Applying the Arithmetic Mean Geometric Mean Inequality

We cannot have x x as negative and y y as positive since 54 x y -54xy will be positive, and so will the other terms, but we can easily see that the expression can attain a negative value, so we can have either x , y x,y as both positive or both negative. Nevertheless, AM-GM is still applicable.

By AM-GM,

x 6 + y 6 + 27 + 27 + 27 + 27 6 x 6 y 6 3 12 6 \frac{x^6+y^6+27+27+27+27}{6} \geq \sqrt[6]{x^6\cdot y^6 \cdot 3^{12}} .

This gives us x 6 + y 6 + 27 + 27 + 27 + 27 6 9 x y \frac{x^6+y^6+27+27+27+27}{6} \geq 9xy or x 6 + y 6 54 x y 108 x^6+y^6 \geq 54xy-108

Subtracting 54 x y 54xy to both sides gives us x 6 + y 6 54 x y 108 x^6+y^6 -54xy \geq -108

So, our answer is 108 \boxed{-108} .

Remember that AM-GM only establishes a lower bound. To show that it is in fact the minimum value you need to show that the expression actually has that value for some x x and y y (in this case the minimum occurs at ( 3 , 3 ) (\sqrt{3},\sqrt{3}) and ( 3 , 3 ) (-\sqrt{3},-\sqrt{3}) ).

Mark C - 5 years ago

Log in to reply

Yes but the lower bound in AM GM is always possible, just set all the terms equal so x^6 =y^6=27

Aadil Bhore - 4 years, 12 months ago

Log in to reply

I'm not sure what you mean. By AM-GM 1 + 2 2 2 \frac{1+2}{2}\geq\sqrt{2} , but clearly that lower bound is not possible. Perhaps you are talking about only a restricted class of applications of AM-GM.

Mark C - 4 years, 12 months ago

Log in to reply

@Mark C See lower bounds are taken into account only during the equality of two or more quantities but not what u have taken..................u can say in this question the value occurs when there is equality between x and y............. I hope i am correct................

Abhisek Mohanty - 4 years, 11 months ago

Very nice solution thanks

Santosh Tripathi - 3 years, 7 months ago

x , y x,y can be negative, then x 6 y 6 6 = x y \sqrt[6]{x^6y^6}=|xy| . So, further explanation needed.

Anindya Biswas - 9 months, 1 week ago

Let me consider: x 6 + y 6 = ( x 2 ) 3 + ( y 2 ) 3 x^6 + y^6 = (x^2)^3 +( y^2)^3

let x 2 = a x^2= a and y 2 = b y^2 = b

Clearly a , b > 0 a,b>0 and hence I can apply AM GM inequality on them

a 3 + b 3 2 a 3 b 3 = 2 ( x y ) 3 a^3 + b^3 \ge 2 \sqrt{a^3b^3} = 2(xy)^3 or 2 ( x y ) 3 2(-xy)^3

Now there arise two conditions :

  1. When x y < 0 xy<0 ( no minimum can be achieved in this case , u may confirm it by differentiation)
  2. When x y > 0 xy >0

Now let x y = t xy = t , using this substitution the equation can be written as

x 6 + y 6 54 x y 2 t 3 54 t x^6 + y^6 -54xy \ge 2t^3 - 54t

Using calculus u may calculate the minimum value of t 3 54 t t^3 - 54t which will turn out to be 108 -108

Alex Hack
Apr 29, 2019

Using AM-GM inequality we have:

x 6 + y 6 54 x y 2 x y 3 54 x y x^6+y^6-54xy\geq 2|xy|^3-54xy .

Let a = x y a=xy . Thus we have to minimise the one variable function f ( a ) = 2 a 3 54 a f(a)=2|a|^3-54a . This can be done very easily using calculus, but I would like to apply AM-GM again, using an approach similar to @Manuel Kahayon .

Consider first the case a > 0 a>0 . I need to add e subtract some (two in this case because we have a a to the power of 3 3 ) positive terms in order to apply the inequality and I would like to get + 54 a +54a when these terms are multiplied together. So I can rewrite the function:

2 a 3 + b 1 + b 2 b 1 b 2 54 a 3 ( 2 a 3 b 1 b 2 ) 1 3 b 1 b 2 54 a 2a^3+b_1+b_2-b_1-b_2-54a\leq 3(2a^3b_1b_2)^{\frac{1}{3}}-b_1-b_2-54a .

Now if b 1 = b 2 = 54 b_1=b_2=54 we have 3 ( 2 5 4 2 a 3 ) 1 3 = 54 a 3(2*54^2a^3)^{\frac{1}{3}}=54a and so we have:

2 a 3 + 54 + 54 108 54 a 108 2a^3+54+54-108-54a\leq -108

When a = 3 > 0 a=3>0 we have affectively f ( 3 ) = 108 f(3)=-108 so 108 -108 is a minimum when a > 0 a>0 .

Moreover, when a 0 a\leq 0 we have f ( a ) 0 f(a)\geq 0 and so 108 -108 is a global minimum of f ( a ) f(a) .

Another beautiful solution. Thank you.

Pi Han Goh - 2 years, 1 month ago
Emmanuel Torres
Feb 8, 2017

Assume x = y, they have a similar relationship, and simple is more efficient. So we have 2x^6 - 54x^2. Take the derivative. You get 12x^5 - 108x. You want this to equal 0. So we have 12x^5 = 108x. 12x^4 = 108. x^4 = 9. x^2 = 3. Plug in x^2 = 3 into 2x^6 - 54x^2. 54 - 162 = -108.

You have only shown that the expression attains a value of -108 when x=y. How do you know that the expression can't take a smaller value when x = y is not true?

Pi Han Goh - 4 years, 4 months ago

I assumed that the minimum value was when x=y. x and y should each have an absolute value greater than 1, so that the expression has potential to be very negative. Assuming that, if x and y are both raised to the 6th power, they will become large positive values, because 6 is an even exponent. In addition, it also makes the equation slightly more negative. For example, plug in 2 cases. Case 1: x = 1, y = 3. Case 2: x = 2, y = 2. Even without calculating, I know that case 2 is more negative, because the product is larger, (1 * 3 = 3 compared to 2 * 2 = 4) and the sum of the exponents is smaller, (1^6 + 3^6 = 730 compared to 2^6 + 2^6 = 128). To minimize the positive value, it is best to make x =y.

Emmanuel Torres - 4 years, 4 months ago

Log in to reply

No, this is not a rigorous argument, note that there is a "-54xy" term as well, so your explanation is incomplete to say the least.

Your solution boils down to "Oh, I guessed the minimum value occurs when x=y". But that is not always the case. Read up Inequalities with strange equality conditions .

Pi Han Goh - 4 years, 4 months ago

Log in to reply

But the eq is a symmetric one.So we can guess that the optimisation occurs when x=y.

Spandan Senapati - 4 years, 3 months ago

Log in to reply

@Spandan Senapati That's a common misconception. Read up Inequalities with strange equality conditions .

Pi Han Goh - 4 years, 3 months ago

Log in to reply

@Pi Han Goh Oh thanks sir...

Spandan Senapati - 4 years, 3 months ago

Can the problem be solved using partial derivative?

Shubam agarwal - 2 years, 4 months ago

Log in to reply

Yes, see second derivative test .

Pi Han Goh - 2 years, 4 months ago

0 pending reports

×

Problem Loading...

Note Loading...

Set Loading...